Three students are sitting on a school bus. Jack is 2 feet directly behind Destiny and 9 feet
directly left of Barbara, Jack makes a paper airplane and throws it to Destiny. Destiny throws
the airplane to Barbara, who throws it back to Jack. How far has the paper airplane traveled?
If necessary, round to the nearest tenth.

Answers

Answer 1

The total distance traveled by the airplane is 15.3 feet.

What is distance formula?

The Pythagorean theorem asserts that the square of the length of the hypotenuse in a right triangle equals the sum of the squares of the lengths of the two legs in a right triangle. This theorem is the basis for the distance formula. In the distance formula, the hypotenuse is the distance between the two points, and the two legs are the differences between the x- and y-coordinates of the two points. Any two locations in a two-dimensional coordinate system can have their distance between them calculated using the formula.

Let us suppose the starting point, that is, the point for jack as (0, 0).

Now, according to the given placements the position of the other students are:

Destiny: (0,2)

Barbara: (-9,2)

Now, using the distance formula we have:

The distance between Jack and Destiny is:

√[(0-0)² + (2-0)²] = √(4) = 2

The distance between Destiny and Barbara is:

√[(-9-0)² + (2-2)²] = √(81) = 9

The distance between Barbara and Jack is:

√[(0-(-9))² + (0-2)²] = √(85)

So the total distance traveled by the paper airplane is:

2 + 9 + √(85) ≈ 15.3 feet

Hence, the total distance traveled by the airplane is 15.3 feet.

Learn more about distance formula here:

https://brainly.com/question/28956738

#SPJ1


Related Questions

Suppose that X is a continuous random variable whose probability density function is given by and for other values of What is the value of C?

Answers

For a continuous random variable, X, with probability density function, [tex]f(x) = \[ \begin{cases}C(4x −2x²)& 0 < x < 2 \\ 0 &otherwise \end{cases} \][/tex], the value of C is equals to [tex]C= \frac{ 3}{8}[/tex].

A continuous random variable has an uncountably infinite number of possible values. The condition for valid pdf of a continuous random variable is [tex]\int_{- \infty }^{ \infty } f(x)dx = 1[/tex]. We have a continuous random variable, X, with probability density function (pdf), f(x), defined as [tex]f(x) = \[ \begin{cases}C(4x −2x²)& 0 < x < 2 \\ 0 &otherwise \end{cases} \][/tex] which is a pointwise function. Since f is a probability density function, we must have [tex]\int_{- \infty }^{ \infty } f(x)dx = 1[/tex], implying that, so, [tex]\int_{-\infty }^{ 0 } f(x)dx + \int_{0 }^{ 2 }f(x)dx + \int_{2}^{\infty } f(x)dx = 1 \\ [/tex]

Now, check the function carefully are plug the values of probability density function. So, [tex]\int_{- \infty }^{ 0 } 0dx + \int_{0 }^{ 2 } C(4x −2x²),dx + \int_{2}^{\infty }0 \ dx = 1 \\ [/tex]

=> [tex] \int_{0 }^{ 2 } C(4x −2x²)dx = 1[/tex]

Using the integration rules,

[tex]C(\int_{0 }^{ 2 }4 x dx − \int_{0 }^{ 2 } 2x²dx) = 1[/tex]

[tex]4C[ \frac{x²}{2}]_{0 }^{ 2 } − C[\frac{2x³}{3}]_{0 }^{ 2 }= 1 [/tex]

= >[tex]C[4 \frac{2²}{2}-0]−C[\frac{2× 2³}{3}-0] = 1[/tex]

=> [tex]8C- \frac{16}{3}C = 1[/tex]

=> [tex] \frac{ 8}{3}C= 1[/tex]

=> [tex]C = \frac{ 3}{8}[/tex]

Hence, required value is [tex]C = \frac{ 3}{8}[/tex].

For more information about probability density function,

https://brainly.com/question/30717978

#SPJ4

Complete question:

Suppose that X is a continuous random variable whose probability density function is given by f (x) =C(4x −2x²), 0<x <2 0, (1) What is the value of C?

-X For the formula at the given point, find the equation of the tangent line 5. y = x’e* at the point (1,1/e) 6. y = (1+2x)10 at x=0 - -

Answers

The equation of the tangent line is y= x/e.

We have function

f(x) = x²[tex]e^{-x[/tex]

We have to find the equation of tangent at the point (1,1 /e)

So, Equation of tangent

dy/dx = - x²[tex]e^{-x[/tex] + 2 [tex]e^{-x[/tex]

Now, at point (1, 1/e)

dy/dx =  - 1²[tex]e^{-1[/tex] + 2 [tex]e^{-1[/tex]

dy/dx= 1/e

Thus, the equation of tangent passing through (1, 1/e)

y- 1/e = 1/e(x-1)

y= x/e - 1/e + 1/e

y= x/e

Learn more about Tangent Equation here:

https://brainly.com/question/14405918

#SPJ4

Who is Thesus in the “Cruel Tribute”?

Answers

Answer:   This tribute was to prevent Minos starting a war after Minos’ son, Androgens, was killed in Athens by unknown assassins during the games. Theseus volunteered to be one of the men, promising to kill the Minotaur and end the brutal tradition.

Step-by-step explanation:

Have a good day!!

the following data represent a random sample of the ages of players in a baseball league. assume that the population is normally distributed with a standard deviation of 1.8 years. find the 95% confidence interval for the true mean age of players in this league. round your answers to two decimal places and use ascending order.

Answers

The 95% confidence interval for the true mean age of players in this baseball league is (27.58, 29.82).

To find the 95% confidence interval, we need to follow these steps:

1. Calculate the sample mean:

(32 + 24 + 30 + 34 + 28 + 23 + 31 + 33 + 27 + 25) / 10 = 287 / 10 = 28.7

2. Determine the standard error of the sample mean:

Standard error = Standard deviation / sqrt(sample size) = 1.8 / sqrt(10) ≈ 0.5698

3. Determine the critical value for the 95% confidence level (using the z-table, since the population standard deviation is known):

Critical value (z-score) ≈ 1.96

4. Calculate the margin of error:

Margin of error = Critical value * Standard error ≈ 1.96 * 0.5698 ≈ 1.1168

5. Find the confidence interval:

Lower limit = Sample mean - Margin of error = 28.7 - 1.1168 ≈ 27.58

Upper limit = Sample mean + Margin of error = 28.7 + 1.1168 ≈ 29.82

So, the 95% confidence interval is (27.58, 29.82), rounded to two decimal places and in ascending order.

Note: The question is incomplete. The complete question probably is: The following data represent a random sample of the ages of players in a baseball league. Assume that the population is normally distributed with a standard deviation of 1.8 years. Find the 95% confidence interval for the true mean age of players in this league. Round your answers to two decimal places and use ascending order. Age: 32, 24, 30,34,28, 23,31,33,27,25.

Learn more about Confidence interval:

https://brainly.com/question/15712887

#SPJ11

Please use formulas and step by stepThe distribution of F, with df in the numerator of 50 and df in the denominator of 20, find the value of F so that the area is:a). from F to the right 0.01, andb). from F to the right 0.05.

Answers

The value of F such that the area is from F to the right 0.05 is 2.231.

The distribution of F with df in the numerator of 50 and df in the denominator of 20 can be denoted as F(50,20).

a) To find the value of F so that the area is from F to the right 0.01, we need to use the inverse F distribution table or calculator. Specifically, we want to find the value of Fα such that P(F > Fα) = α = 0.01.

Using the table or calculator, we find that Fα = 2.911. Therefore, the value of F such that the area is from F to the right 0.01 is 2.911.

b) To find the value of F so that the area is from F to the right 0.05, we again need to use the inverse F distribution table or calculator. Specifically, we want to find the value of Fα such that P(F > Fα) = α = 0.05.

Using the table or calculator, we find that Fα = 2.231.

Know more about denominator here:

https://brainly.com/question/7067665

#SPJ11

find the equation of straight line passing through (5,-5) and (-3,7)​

Answers

The equation of the straight line passing through the points (5, -5) and (-3, 7) is 3x + 27 - 5 = 0.

Finding the equation of a straight line:

To find the equation of a straight line passing through two given points, we use the point-slope form of the equation of a line:

                             =>  (y - y₁) = m(x - x₁)

Where (x₁, y₁) is one of the given points, m is the slope of the line, and (x, y) are the coordinates of any other point on the line.

Here we have

The straight  line passing through (5,-5) and (-3,7)​

From the given points the slope of the line can be found as follows

m = (y₂ - y₁)/(x₂ - x₁) = (7 - (-5))/(-3 - 5) = 12/-8 = - 3/2

Using the above formula,

=> y - (-5) = -3/2 (x - 5)

=> y + 5 = -3x/2 + 15/2

=> 2(y + 5) = - 3x + 15

=> 2y + 10 = -3x + 15

=> 3x + 27 - 5 = 0

Therefore,

The equation of the straight line passing through the points (5, -5) and (-3, 7) is 3x + 27 - 5 = 0.  

Learn more about Straight Line at

https://brainly.com/question/24955512

#SPJ1

A random sample of likely voters showed that 62​% planned to vote for Candidate​ X, with a margin of error of 4 percentage points and with​ 95% confidence.
a. Use a carefully worded sentence to report the​ 95% confidence interval for the percentage of voters who plan to vote for Candidate X.

Answers

The random sample of likely voters, we can say with 95% confidence that the percentage of voters who plan to vote for Candidate X falls within the interval of 58% to 66%.

To repeat the sampling process multiple times.

95% of the intervals calculated would contain the true population proportion of voters who plan to vote for Candidate X.

The margin of error of 4 percentage points tells us that if we were to conduct the same survey multiple times.

The sample proportion would vary within a range of plus or minus 4 percentage points from the true population proportion.

It is important to note that this confidence interval only applies to the specific sample of likely voters that was surveyed and may not necessarily reflect the views of the entire population.

Nevertheless, this interval can provide a useful estimate for predicting the likely outcome of an election and can be used by campaigns to strategize their messaging and target certain demographics.

For similar questions on Random sample

https://brainly.com/question/24466382

#SPJ11

A poll is taken in which 390390 out of 550550 randomly selected voters indicated their preference for a certain candidate.

(a) Find a 9595% confidence interval for pp.

≤p≤≤p≤

(b) Find the margin of error for this 9595% confidence interval for pp.

(c) Without doing any calculations, indicate whether the margin of error is larger or smaller or the same for an 80% confidence interval.
A. larger
B. smaller
C. same

Answers

(a) To find a 95% confidence interval for p, we use the formula:

p ± Z * sqrt(p * (1-p) / n)

where p = 390/550 (sample proportion), Z = 1.96 (for a 95% confidence interval), and n = 550 (sample size).

p = 390/550 ≈ 0.7091

Confidence interval = 0.7091 ± 1.96 * sqrt(0.7091 * (1-0.7091) / 550)
≈ 0.7091 ± 0.0425

So, the 95% confidence interval is 0.6666 ≤ p ≤ 0.7516.

(b) The margin of error for this 95% confidence interval is:

1.96 * sqrt(0.7091 * (1-0.7091) / 550) ≈ 0.0425

(c) Without doing any calculations, the margin of error for an 80% confidence interval would be:

B. smaller

This is because a lower confidence level results in a smaller margin of error.

In frequentist statistics, a confidence interval (CI) is a range of estimates for an unknown parameter. A confidence interval is computed at a designated confidence level; the 95% confidence level is most common, but other levels, such as 90% or 99%, are sometimes used.[1][2] The confidence level represents the long-run proportion of CIs (at the given confidence level) that theoretically contain the true value of the parameter. For example, out of all intervals computed at the 95% level, 95% of them should contain the parameter's true value.

Learn more about confidence interval here:

https://brainly.com/question/24131141

#SPJ11

I need with process

what is 25% of 530?


53% of what number is 384


what % of 369 is 26


43 is 31% of what number


what is 74% of 44


105 is 42% of what number

Answers

Answer:

25% of 530 is 132.5

53% of 724.53 is 384

7.05% of 369 is 26

43 is 31% of 138.7

74% of 44 is 32.56

105 is 42% of 250

Step-by-step explanation:

(25/100)*530 = 132.5

(384*100)/53 = 384

(26/369)*100 = 7.05%

(43*100)/31 = 138.7

(74/100)*44 = 32.56

(105*100)/42 250

Mina takes her test at 1:15 pm. What will time will it be 135 minutes after 1:15 pm?

Answers

Answer: 3:30

Step-by-step explanation:

The adjusted R squared is used when we are doing multiple regression (i.e more than one independent variable) True False

Answers

The adjusted R squared is used when we are doing multiple regression

True.

In multiple regression analysis, there are usually several independent variables that are used to predict a single dependent variable. The adjusted R squared is a statistical measure that is commonly used to assess the goodness of fit of a multiple regression model. It is a modified version of the R squared statistic, which represents the proportion of variance in the dependent variable that can be explained by the independent variables.

The adjusted R squared is useful when working with multiple regression models because it takes into account the number of independent variables included in the model. As the number of independent variables increases, the R squared value can increase even if the model does not fit the data well. The adjusted R squared adjusts for this by penalizing the R squared value for every additional independent variable included in the model.

The adjusted R squared is therefore a more reliable measure of the goodness of fit of a multiple regression model than the R squared statistic alone. It helps to ensure that the model is not overfitting the data and that the independent variables included in the model are truly contributing to the prediction of the dependent variables.

for such more questions on variables

https://brainly.com/question/28248724

#SPJ11

provide two potencial examples of a sequence {an} ♾ n=1 thata. Convergesb. Diverges

Answers

An example of a sequence that a. Converges is an = 1/n and that b. Diverges an = n

The two potential examples of a sequence {an} with n=1 to infinity that converges and diverges:

a. Converges: A sequence that converges is one where the terms approach a finite limit as n goes to infinity. An example is the sequence an = 1/n. As n increases, the terms get smaller and approach 0, which is the limit.

b. Diverges: A sequence that diverges is one where the terms do not approach any finite limit as n goes to infinity. An example is the sequence an = n. As n increases, the terms also increase without bounds, so the sequence diverges.

Learn more about  sequences and series: https://brainly.com/question/26263191

#SPJ11

Probability and States Class Activity 1 5. The continuous random variable X takes value in the interval 0

Answers

the probability that X takes a value between 1 and 1.5 is approximately 0.1172

Probability is a branch of mathematics in which the chances of experiments occurring are calculated. The probability density function (PDF) of a continuous random variable X that takes values in the interval [0, 2] is given by:

f(x) = kx(2-x), where 0 <= x <= 2

To find the value of k, we need to use the fact that the integral of the PDF over the entire interval [0, 2] is equal to 1 (since the total probability of all possible outcomes must equal 1):

∫[0,2] f(x) dx = ∫[0,2] kx(2-x) dx = 1

Expanding the integral and solving for k, we get:

k ∫[0,2] x(2-x) dx = 1

k [∫[0,2] 2x dx - ∫[0,2] x^2 dx] = 1

k [x^2 - (1/3)x^3] from 0 to 2 = 1

k (4/3) = 1

k = 3/4

Therefore, the PDF of X is given by:

f(x) = (3/4)x(2-x), where 0 <= x <= 2

We can now find the probability that X takes a value between 1 and 1.5 by integrating the PDF over that interval:

P(1 <= X <= 1.5) = ∫[1,1.5] (3/4)x(2-x) dx

= (3/4) ∫[1,1.5] x(2-x) dx

= (3/4) [(2x^2/2 - x^3/3) from 1 to 1.5]

≈ 0.1172

Therefore, the probability that X takes a value between 1 and 1.5 is approximately 0.1172

learn about probability,

https://brainly.com/question/13604758

#SPJ11

So I have to find the angle QST and the numbers are 2x + 18 and 8x + 12 and it has to equal 40, but I don't know how to solve it

Answers

The angle QRS is 25 degrees and the angle STR is 40 degrees.

How to calculate the angle

QST + QRS + STR = 180

We also know that QST = 40, and we can express the other two angles in terms of x using the given expressions:

QRS = 2x + 18

STR = 8x + 12

Substituting these values into the first equation, we get:

40 + (2x + 18) + (8x + 12) = 180

Simplifying and solving for x:

10x + 35 = 70

10x = 35

x = 3.5

QRS = 2x + 18 = 2(3.5) + 18 = 25

STR = 8x + 12 = 8(3.5) + 12 = 40

Leans more about angle on

https://brainly.com/question/25770607

#SPJ4

A marketing research company is interested in determining whether there is a significant difference between the number of customers who prefer Brand A and the number of customers who prefer Brand B.Customer Prefers Brand A1 Yes2 No3 Yes4 Yes5 Yes6 No7 Yes8 Yes9 NoNote by completing this case study, you will have conducted an appropriate hypothesis test.Find the p-value using Excel. Show your Excel command and your final answer, rounded to 4 decimal places. Do not round any values until you reach your final answer.[1 mark] Based on this p-value, do you reject the null hypothesis (answer "yes" or "no", with no additional words)?In one sentence, conclude in the context of the original question.4. Note the sample size here is relatively small. Name one tactic you might use to encourage more people to fill out the survey. Name a possible problem of using such a tactic.

Answers

To determine if there is a significant difference between the number of customers who prefer Brand A and those who prefer Brand B, an appropriate hypothesis test can be conducted by using a two-sample proportion z-test.


To conduct an appropriate hypothesis test, use Excel's chi-square test function to find the p-value. In this case, create a table with the counts for Brand A (6 Yes, 3 No) and Brand B (assuming the opposite, 3 Yes, 6 No).

In Excel, use the command =CHISQ.TEST(A1:B1, A2:B2), where A1:B1 contains the counts for Brand A (6, 3) and A2:B2 contains the counts for Brand B (3, 6). The p-value calculated is 0.0763.

Based on this p-value, the answer is no, you do not reject the null hypothesis.

In conclusion, there is no significant difference between the number of customers who prefer Brand A and those who prefer Brand B.

The p-value can be found using Excel with the command "=1- NORM.S.DIST(Z test statistic, TRUE)" where the test statistic is calculated by subtracting the two sample proportions and dividing the result by the standard error of the difference between proportions.

Based on the p-value obtained, if it is less than the significance level (usually 0.05), we can reject the null hypothesis and conclude that there is a significant difference between the two brands.

One tactic to encourage more people to fill out the survey is to offer an incentive or reward for participating, such as a discount on their next purchase.

However, a possible problem of using such a tactic is that it may attract respondents who are not genuinely interested in the brands or who may not represent the target market, leading to biased results

Visit here to learn more about Null Hypothesis:

brainly.com/question/4436370

#SPJ11

Subtract. Write your answer in simplest form. 7 1/4- 4 5/12

A. 2 5/6

B. 3, 1/6

C. 3, 1/2

D. 2, 2/3

Answers

To subtract 4 5/12 from 7 1/4, we need a common denominator of 48. The simplified result is 2 5/6.

What are equations?

An equation is a mathematical statement that states that two expressions are equal. It consists of two sides, left and right, separated by an equal sign (=). Equations can include variables, which are symbols that represent unknown values or values that can vary. Solving an equation involves finding the value of the variable that makes the equation true.

According to the given information:

To subtract 4 5/12 from 7 1/4, we need to have a common denominator.

Multiplying the denominators 4 and 12, we get 48 as the least common denominator.

Converting the fractions to have a denominator of 48:

7 1/4 = 7 * 48/48 + 12/48 = 336/48 + 12/48 = 348/48

4 5/12 = 4 * 48/48 + 20/48 = 192/48 + 20/48 = 212/48

Subtracting the second fraction from the first:

7 1/4 - 4 5/12 = 348/48 - 212/48 = 136/48

Simplifying the result by dividing both numerator and denominator by their greatest common factor, which is 8:

136/48 = 17/6

the answer is (A) 2 5/6.

Therefore, To subtract 4 5/12 from 7 1/4, we need a common denominator of 48. The simplified result is 2 5/6.

To know more about equations visit:

brainly.com/question/22688504

#SPJ1

To subtract [tex]4\frac{5}{12}[/tex] from [tex]7\frac{1}{4}[/tex], we need a common denominator of 48. The simplified result is [tex]2\frac{5}{6}[/tex]

What are equations?

An equation is a mathematical statement that states that two expressions are equal. It consists of two sides, left and right, separated by an equal sign (=). Equations can include variables, which are symbols that represent unknown values or values that can vary. Solving an equation involves finding the value of the variable that makes the equation true.

According to the given information:

To subtract  from 7 1/4, we need to have a common denominator.

Multiplying the denominators 4 and 12, we get 48 as the least common denominator.

Converting the fractions to have a denominator of 48:

7 1/4 = 7 * 48/48 + 12/48 = 336/48 + 12/48 = 348/48

4 5/12 = 4 * 48/48 + 20/48 = 192/48 + 20/48 = 212/48

Subtracting the second fraction from the first:

7 1/4 - 4 5/12 = 348/48 - 212/48 = 136/48

Simplifying the result by dividing both numerator and denominator by their greatest common factor, which is 8:

136/48 = 17/6

the answer is (A) 2 5/6.

Therefore, To subtract 4 5/12 from 7 1/4, we need a common denominator of 48. The simplified result is 2 5/6.

To know more about equations visit:

brainly.com/question/22688504

#SPJ1

Find fx,fy and fz. f(x, y, z) = tan^-1 (1/xy²4)

Answers

The values of the function are,

⇒ fx = -y⁻²/(1 + (1/x²y⁴)), fy = -2xy⁻³/(1 + (1/x²y⁴)), and fz = 0.

Now, let's find the partial derivative of f(x, y, z) with respect to x, y, and z as:

f (x, y, z) = tan ⁻¹ (1/x²y⁴)

Hence, We get;

⇒ ∂f/∂x = -y⁻²/(1 + (1/x²y⁴))

⇒ ∂f/∂y = -2xy⁻³/(1 + (1/x²y⁴))

⇒ ∂f/∂z = 0

Therefore, the gradient of f(x, y, z) is:

∇f = (∂f/∂x)i + (∂f/∂y)j + (∂f/∂z)k = (-y⁻²/(1 + (1/x²y⁴)))i + (-2xy⁻³/(1 + (1/x²y⁴)))j + 0k

So, We get;

fx = -y⁻²/(1 + (1/x²y⁴)), fy = -2xy⁻³/(1 + (1/x²y⁴)), and fz = 0.

Learn more about the function visit:

https://brainly.com/question/11624077

#SPJ4

What is the third quartile, Q3, of the data represented by the box plot? 0 12. 5 20 65 143. 75

Answers

The box plot for the given dataset shows that the third quartile (Q3) is equal to 143.75.

Box plots are a graphical representation of a set of data that shows the distribution of the data and its various quartiles. The box plot displays the minimum value, first quartile (Q1), median (Q2), third quartile (Q3), and maximum value of a dataset.

Now, let's take a look at the box plot for the given data: 0 12.5 20 65 143.75. The box plot consists of a box that represents the middle 50% of the data (i.e., from Q1 to Q3). The line inside the box represents the median (Q2). The lower whisker represents the minimum value, and the upper whisker represents the maximum value.

To find the third quartile (Q3) from the box plot, we need to look at the right-hand side of the box. We can see that the box ends at approximately 143.75, which is also the maximum value of the dataset. Therefore, Q3 is equal to 143.75.

To know more about boxplot here

https://brainly.com/question/12992903

#SPJ4

Data: 1 bicycle, 1 boat, 25 buses, 192 cars, 1 other, 5 rails, 17 walks, 8 blanks
In this sample, what is the sample proportion of students who travel to school by car?
What is the standard error? You may use the simple formula.
Calculate the simple version of the 95% confidence interval and interpret this CI.
In order to use the "simple" formula, what extra qualification must be met?

Answers

In order to use the simple formula, the sample size should be large enough to ensure that both the sample proportion and the complement of the sample proportion (1 - sample proportion) are at least 5. In this case, 192 and 58 are both greater than 5, so the qualification is met.

The sample proportion of students who travel to school by car is 192/250 or 0.768.

To calculate the standard error using the simple formula, we use the formula:

Standard Error = Square Root [(Sample Proportion * (1 - Sample Proportion)) / Sample Size]

Plugging in the values, we get:

Standard Error = Square Root [(0.768 * (1 - 0.768)) / 250]
= 0.034

To calculate the simple version of the 95% confidence interval, we use the formula:

CI = Sample Proportion ± (Z * Standard Error)

Where Z is the z-score associated with the desired level of confidence. For a 95% confidence interval, Z is 1.96.

Plugging in the values, we get:

CI = 0.768 ± (1.96 * 0.034)
= 0.701 to 0.835

Interpreting this CI, we can say with 95% confidence that the true proportion of students who travel to school by car in the population lies between 0.701 and 0.835.

Know more about sample proportion here:

https://brainly.com/question/29912751

#SPJ11

2) Answer the following optimization problems systematically: d. Find the radius and height of the right circular cylinder of largest volume that can be inscribed in a right circular cone with radius 4 cm and height 3 cm.

Answers

The radius and height of the right circular cylinder of largest volume inscribed in a right circular cone with radius 4 cm and height 3 cm are approximately 2.667 cm and 1.333 cm, respectively.


1. Let r and h be the radius and height of the cylinder.


2. Use similar triangles: r/R = h/H, where R and H are the radius and height of the cone (4 cm and 3 cm).


3. Obtain the expression for the volume of the cylinder: V = πr²h.


4. Substitute the ratio from step 2: V = π(r³/3).


5. Differentiate the volume function with respect to r: dV/dr = πr².


6. Set dV/dr to 0 and solve for r: r = 2.667 cm.


7. Substitute r back into the ratio from step 2: h = 1.333 cm.

To know more about right circular cylinder click on below link:

https://brainly.com/question/30517598#

#SPJ11

orwrite a system of equations to describe the situation below, solve using substitution, and fill in the blanks.austen wants to take group fitness classes at a nearby gym, but needs to start by selecting a membership plan. with the first membership plan, austen can pay $47 per month, plus $3 for each group class he attends. alternately, he can get the second membership plan and pay $41 per month plus $4 per class. if austen attends a certain number of classes in a month, the two membership plans end up costing the same total amount. what is that total amount? how many classes per month is that?

Answers

Each membership plan costs $65 if Austen takes 6 classes per month.

Let's write a system of equations to describe the situation, solve it using substitution, and fill in the blanks.

Let x be the number of classes Austen takes per month, and y be the total cost of the membership plan.

For the first membership plan, the equation is:

y = 47 + 3x

For the second membership plan, the equation is:

y = 41 + 4x

Since both plans cost the same total amount, we can set the equations equal to each other and solve for x:

47 + 3x = 41 + 4x

In order to find x, follow these steps:

1. Subtract 3x from both sides:

47 = 41 + x

2. Subtract 41 from both sides:

6 = x

3. Now we know that Austen takes 6 classes per month. Let's plug the value of x back into one of the equations to find the total cost (y). We can use the first equation:

y = 47 + 3(6)

4. Multiply 3 by 6:

y = 47 + 18

5. Add 47 and 18:

y = 65

Hence, if Austen takes 6 classes per month then  each membership plan costs $65

Note: The question is incomplete. The complete question probably is: Austen wants to take group fitness classes at a nearby gym, but needs to start by selecting a membership plan. With the first membership plan, Austen can pay $47 per month, plus $3 for each group class he attends. alternately, he can get the second membership plan and pay $41 per month plus $4 per class. If Austen attends a certain number of classes in a month, the two membership plans end up costing the same total amount. What is that total amount? How many classes per month is that?

Each membership plan costs $_____ if Austen takes ____ classes per month.

Learn more about Cost:

https://brainly.com/question/19104371

#SPJ11

A group of 55 bowlers showed that their average score was 190 with a population standard deviation of 8. Find the 99% confidence interval of the mean score of all bowlers.

Answers

We can be 99% confident that the true mean score of all bowlers falls within the interval of (187.224, 192.776).

To find the 99% confidence interval of the mean score of all bowlers, we can use the formula:

CI = x ± z×(σ/√n)

where x is the sample mean (190), σ is the population standard deviation (8), n is the sample size (55), and z is the z-score associated with the desired confidence level (99%).

We can find the z-score using a standard normal distribution table or a calculator, which gives us a value of 2.576.

Substituting the values into the formula, we get:

CI = 190 ± 2.576×(8/√55)
CI = 190 ± 2.576×(1.077)
CI = 190 ± 2.776
CI = (187.224, 192.776)

Therefore, we can be 99% confident that the true mean score of all bowlers falls within the interval of (187.224, 192.776).

To learn more about interval here:

brainly.com/question/13708942#

#SPJ11

At the end of each quarter, $3,500 is placed in an annuity that earns 8% compounded quarterly. Find the future value in ten years.

Answers

On solving the provided question ,we can say that As a result, after 10 sequence years, the annuity's future value will be $413,583.88.

what is a sequence?

A sequence is a grouping of "terms," or integers. Term examples are 2, 5, and 8. Some sequences can be extended indefinitely by taking advantage of a specific pattern that they exhibit. Use the sequence 2, 5, 8, and then add 3 to make it longer. Formulas exist that show where to seek for words in a sequence. A sequence (or event) in mathematics is a group of things that are arranged in some way. In that it has components (also known as elements or words), it is similar to a set. The length of the sequence is the set of all, possibly infinite, ordered items. the action of arranging two or more things in a sensible sequence.

We may utilise the calculation for the future value of an annuity to resolve this issue:

FV is equal to P * ((1 + r/n)(n*t) - 1) / (r/n).

where:

Future Value (FV)

P = periodic payment ($3,500 in this example).

(8%) is the yearly interest rate.

Since interest is compounded quarterly, n equals the number of times per year that interest is compounded.

(10) T = number of years

When we enter the values, we obtain:

FV = 3500 * ((1 + 0.08/4)^(4*10) - 1) / (0.08/4)

FV equals 3500 * (1.0240 - 1) / 0.02 FV equals 3500 * 118.1668 FV equals 413583.88

As a result, after 10 years, the annuity's future value will be $413,583.88.

To know more about sequence visit:

https://brainly.com/question/21961097

#SPJ1

Question 10 Question 1 (5+5+5 points): Not yet answered a) Evaluate s, dx 1 3 (x-2) 3/2 x2 Marked out of 15.00 Answer: P Flag question b) Is the following integral convergent or divergent? (Write C or

Answers

The given integral 1 3 (x-2)³/² +x² is convergent.

The integral can be written as ∫(x-2)³/² dx from 1 to 3 plus ∫x² dx from 1 to 3. The first integral can be solved using the substitution u=x-2, which gives us ∫u³/² du from 0 to 1.

This integral evaluates to 2/5. The second integral is a simple polynomial integral which evaluates to 26. Therefore, the overall value of the given integral is 2/5+26= 26.4, which is a finite value. Hence, the integral is convergent.

To evaluate the given integral, we first need to check whether it is convergent or divergent. We can do this by checking the limit of the integral as the limit of the upper and lower bounds of the integral approaches infinity. If the limit exists and is a finite number, the integral is convergent, else it is divergent.

In this case, we have a definite integral from 1 to 3, so we don't need to worry about infinity. We split the integral into two parts, and solve them individually.

The first integral involves a square root, so we can use substitution to simplify it. The second integral is a polynomial integral which is easy to solve. Adding the values of the two integrals, we get a finite value, which indicates that the given integral is convergent.

To know more about integral click on below link:

https://brainly.com/question/18125359#

#SPJ11

The accompanying Automobile Options dataset provides data on options ordered together for a particular model of automobile. Consider the following rules. Rule 1: If Fastest Engine, then 3 Year Warranty Rule 2: If Faster Engine and 16-inch Wheels, then Traction Control Compute the support, confidence, and lift for each of these rules. Click the icon to view the Automobile Options data. Compute the support, confidence, and lift for Rule 1. The support is The confidence is The lift is (Round to three decimal places as needed.)

Answers

Lift (Fastest Engine → 3 Year Warranty) = Confidence (Fastest Engine → 3 Year Warranty) / Support (3-Year Warranty) for the automobile.

To calculate support, confidence, and lift for Rule 1, follow these steps:

Step 1: Calculate support for Rule 1
Support is the probability of both events (Fastest Engine and 3-Year Warranty) occurring together. To calculate support, divide the number of instances where both events occur by the total number of instances in the dataset.

Support (Fastest Engine → 3 Year Warranty) = (Number of instances with Fastest Engine and 3-Year Warranty) / (Total instances in the dataset)

Step 2: Calculate confidence for Rule 1
Confidence is the probability of 3-Year Warranty, given Fastest Engine. To calculate confidence, divide the number of instances where both events occur by the number of instances where Fastest Engine occurs.

Confidence (Fastest Engine → 3 Year Warranty) = (Number of instances with Fastest Engine and 3-Year Warranty) / (Number of instances with Fastest Engine)

Step 3: Calculate lift for Rule 1
Lift is the ratio of confidence to the support of the event being predicted (3-Year Warranty). To calculate lift, divide the confidence of the rule by the support of 3-Year Warranty.

Lift (Fastest Engine → 3 Year Warranty) = Confidence (Fastest Engine → 3 Year Warranty) / Support (3-Year Warranty)

Make sure to round your answers to three decimal places.

Note: To provide the exact numerical values for support, confidence, and lift, the specific data from the Automobile Options dataset is needed. The steps above outline the process of how to calculate these values.


Learn more about automobile here:

https://brainly.com/question/17141399

#SPJ11

on a certain sum of moneylent out at 20 %per annum for 1 and a half year the compound interest reconked yearly and and recokned yearly half is 178.75 . find the sum

Answers

The sum of moneylent out at interest 20 %per annum for 1 and a half year the compound interest reconked yearly and and recokned yearly half is 178.75 is 16250.

Let sum be p. Here r = 20%, n =3/2 yrs = 1 1/2 yrs.

When compounded yearly i.e. A.

A = p(1+r/100) * [1+{1/2 r}/100)]

= p(1+20/100) * [1+{1/2 *20}/100]

= p x 6/5 x 11/10 = 33p/25

Compound interest = A - p

= 33p/25 - p

= 8p/25

Now when compounded half yearly, then

A = p[1+(1/2 x r)/100]ⁿ*²

= p[1+(1/2 x 20)/100]⁽³/²⁾*²

= p[11/10]³

= 1331p/1000

Compound interest = 1331p/1000 - p = 331p/1000.

Now as per questions,

331p/1000 - 8p/25 = 178.75

p x 11/1000 = 178.75

p = 178.75 x 1000/11

p = 16250

Hence, the sum of moneylent out at interest 20 %per annum for 1 and a half year the compound interest reconked yearly and and recokned yearly half is 178.75 is 16250.

To know more about interest check the below link:

https://brainly.com/question/25720319

#SPJ1

Solve y=f(x)​ for x . Then find the input when the output is 2.

f of x is equal to 1 half x squared minus 7

Answers

So, the inputs for which the output of function f(x) is 2 are x = 3√2 or x = -3√2.

what is equation?

an equation is a mathematical statement that asserts the equality of two expressions. it typically consists of two sides, the left-hand side and the right-hand side, separated by an equal sign (=). the expressions on both sides can contain variables, constants, operations, and functions, and the equation is usually solved by finding the values of the variables that make both sides of the equation equal to each other. equations can be used to model real-world phenomena, analyze data, and solve problems in various fields such as physics, engineering, finance, and statistics.

To solve for x when [tex]y = f(x) = 1/2 x^2 - 7[/tex], we can set y to 2 and solve for x:

[tex]2 = 1/2 x^2 - 7[/tex]

Adding 7 to both sides, we get:

[tex]9 = 1/2 x^2[/tex]

Multiplying both sides by 2, we get:

[tex]18 = x^2[/tex]

Taking the square root of both sides (remembering to consider both the positive and negative roots), we get:

x = ±√18 = ±3√2

So, the inputs for which the output of f(x) is 2 are x = 3√2 or x = -3√2.

To know more about function  visit:

https://brainly.com/question/12431044

#SPJ1

Find y subject to the given conditions. y'' = 24x, y''(0) = 10, y'(O)=5, and y(0) = 3 y(x) = (Simplify your answer. Do not factor.)

Answers

The solution to the given differential equation with the given initial conditions is y = 4x^3 + 5x + 3.

To solve for y, we need to integrate the given differential equation twice with respect to x, using the initial conditions to determine the constants of integration.

Integrating y'' = 24x once gives us y' =[tex]12x^2 + C1,[/tex] where C1 is the constant of integration. Using the condition y'(0) = 5, we can solve for C1 as follows:

y'(0) = [tex]12(0)^2 + C1[/tex]

5 = C1

So, we have y' =[tex]12x^2 + 5.[/tex]

Integrating y' =[tex]12x^2 + 5[/tex] once more gives us y =[tex]4x^3 + 5x + C2[/tex], where C2 is the constant of integration. Using the condition y(0) = 3, we can solve for C2 as follows:

y(0) = [tex]4(0)^3 + 5(0) + C2[/tex]

3 = C2

So, we have y =[tex]4x^3 + 5x + 3.[/tex]

Learn more about differential equation here:

https://brainly.com/question/14620493

#SPJ11

Which report of risk reduction conveys a more significant treatment effect?
a. Relative
b. Absolute
c. Random
d. Qualitative

Answers

C is the correct answer

Gabriel kicks a football. Its height in feet is given by h(t) = -16t² + 88t where t

represents the time in seconds after kick. What is the appropriate domain for this

situation?

Answers

The domain of the function h(t) = -16t² + 88t is equal to [0 , 5 ].

Function is equal to,

h(t) = -16t² + 88t

Where 't' represents the time in seconds after kick

The domain of a function is the set of all possible values of the independent variable for which the function is defined.

Only independent variable is t.

And there are no restrictions on its value.

Since the function represents the height of a football in feet.

The domain should be restricted to the time when the ball is in the air.

From the time of the kick until the time when the ball hits the ground.

The ball hits the ground when its height is 0.

So, the function h(t) = 0

Solve for t to get the time when the ball hits the ground,

⇒ -16t² + 88t = 0

⇒ -16t(t - 5.5) = 0

⇒ t = 0 or t = 5.5

The ball is kicked at t = 0.

So the appropriate domain for this situation is,

0 ≤ t ≤ 5.5

Therefore, the appropriate domain of the function h(t) is for all values of t between 0 and 5.5 seconds (inclusive).

learn more about domain here

/brainly.com/question/30944862

#SPJ4

Other Questions
The squad to the left of the base squad takes one more 30-inch step to the front with the left foot; execute right oblique toward the base squad for one step and then steps 30 inches back to the original front. The squad then begins to half step. What command is this an example of? Validate your answer by comparing the ratio of carbon atoms to oxygen atoms in the chemical formulas of glucose (C6H12O6) versus corona dioxide (CO2). To summarize the central idea in a text, readers should identiflittle-known facts about the topic.the author's opinion about the topic.other readers' opinions about the topic.O scientific facts about the topic. The rite of the pool, which takes a few seconds is what? Teachers should review old math concepts before building upon them.True False . begin by finding the effects of the fiscal change on thelabor market. how does the effect of the permanentincrease in government purchases of 100 compare withthe effect of a temporary increase in purchases of 100? ___________________ is the landmass that existed between Asia and North America during the last Ice Age.-Beringia -Coastal Canada-Antactic-Arctic three processes required for successful reproduction of angiospermophyta Do you think the Greeks would value verse more than poetry or vice-versa? which of the following compounds exhibit hydrogen bonding? select all that apply. group of answer choices hcl ch4 h2te ch3oh hi hf h3coch3 ch2f2 nh3 ash3 h2o All the following absorb more radiation than they reflect except:A. OzoneB. AsphaltC. Water vaporD. Snow cover Let f(x) = xex 2 . a. [4 points] Find the Taylor series of f(x) centered at x = 0. Be sure to include the first 3 nonzero terms and the general term. Solution: We can use the Taylor series of e y to find the Taylor series for e x 2 by substituting y = x 2 . e x 2 = X[infinity] n=0 (x 2 ) n n! = 1 + (x 2 ) + (x 2 ) 2 2! + + (x 2 ) n n! + Therefore the Taylor series of xex 2 is xex 2 = X[infinity] n=0 (1)nx 2n+1 n! = x x 3 + x 5 2! + + (1)nx 2n+1 n! + b. [2 points] Find f (15)(0). Solution: We know that f (15)(0) 15! will appear as the coefficient of the degree 15 term of the Taylor series. Using part (a), we see that the degree 15 term has coefficient 1 7! . Therefore f (15)(0) = 15! 7! = 259, 459, 200 Find X (x=4, need work) A man is a political animan therefore, plato, Aristotle and Socrates are political animals. is orgument or not? Why do you seem to be a good fit for the JMP? invasion of basement membrane occurs due to The tread life of a particular brand of tire is a random variable best described by a normal distribution with a mean of 61,000 miles and a standard deviation of 2100 miles. What is the probability a certain tire of this brand will last between 60,010 miles and 58,580 miles? If F = 40 N and M = 2.0 kg, what is the magnitude of the acceleration of the suspended object? All surfaces are frictionless. 1) 1.2 m/s2 2) 2.0 m/s2 3) 1.5 m/s2 4) 2.5 m/s2 5) 5.6 m/s2 Learn something about Trichuris, Nematodirus, and Eimeria macusaniensis, parasites that build up when lots of camelids are kept in small paddocks. Erb's palsy is an injury of?what are causes?